Difference between revisions of "2019 AMC 10A Problems/Problem 25"
(→Solution 1) |
(→Solution 1) |
||
Line 17: | Line 17: | ||
<cmath>\frac{(n^2-1)!}{(n!)^n}=\frac{(n^2)!}{(n!)^{n+1}}\cdot\frac{n!}{n^2}</cmath> | <cmath>\frac{(n^2-1)!}{(n!)^n}=\frac{(n^2)!}{(n!)^{n+1}}\cdot\frac{n!}{n^2}</cmath> | ||
− | is an integer if <math> | + | is an integer if <math>n! \n^2</math>, or in other words, if <math>(n-1)! \n</math> is an integer. This condition is false precisely when <math>n=4</math> or <math>n</math> is prime, by [[Wilson's Theorem]]. There are <math>15</math> primes between <math>1</math> and <math>50</math>, inclusive, so there are <math>15 + 1 = 16</math> terms for which |
<cmath>\frac{(n^2-1)!}{(n!)^{n}}</cmath> | <cmath>\frac{(n^2-1)!}{(n!)^{n}}</cmath> |
Revision as of 14:21, 21 August 2021
- The following problem is from both the 2019 AMC 10A #25 and 2019 AMC 12A #24, so both problems redirect to this page.
Problem
For how many integers between and , inclusive, is an integer? (Recall that .)
Solution
Solution 1
The main insight is that
is always an integer. This is true because it is precisely the number of ways to split up objects into unordered groups of size . Thus,
is an integer if $n! \n^2$ (Error compiling LaTeX. Unknown error_msg), or in other words, if $(n-1)! \n$ (Error compiling LaTeX. Unknown error_msg) is an integer. This condition is false precisely when or is prime, by Wilson's Theorem. There are primes between and , inclusive, so there are terms for which
is potentially not an integer. It can be easily verified that the above expression is not an integer for as there are more factors of in the denominator than the numerator. Similarly, it can be verified that the above expression is not an integer for any prime , as there are more factors of p in the denominator than the numerator. Thus all values of n make the expression not an integer and the answer is .
Solution 2
We can use the P-Adic Valuation (more info could be found here: Mathematicial notation) of n to solve this problem (recall the P-Adic Valuation of 'n' is denoted by and is defined as the greatest power of some prime 'p' that divides n. For example, or .) Using Legendre's formula, we know that :
Seeing factorials involved in the problem, this prompts us to use Legendre's formula where n is a power of a prime.
We also know that , . Knowing that if , we have that :
and we must find all n for which this is true.
If we plug in , by Legendre's we get two equations:
And we also get :
But we are asked to prove that which is false for all 'n' where n is prime.
Now we try the same for , where p is a prime. By Legendre we arrive at:
and
Then we get:
Which is true for all primes except for 2, so doesn't work. It can easily be verified that for all where is an integer greater than 2, satisfies the inequality :
Therefore, there are 16 values that don't work and values that work.
~qwertysri987
See Also
Video Solution by Richard Rusczyk: https://www.youtube.com/watch?v=9klaWnZojq0
2019 AMC 10A (Problems • Answer Key • Resources) | ||
Preceded by Problem 24 |
Followed by Last Problem | |
1 • 2 • 3 • 4 • 5 • 6 • 7 • 8 • 9 • 10 • 11 • 12 • 13 • 14 • 15 • 16 • 17 • 18 • 19 • 20 • 21 • 22 • 23 • 24 • 25 | ||
All AMC 10 Problems and Solutions |
2019 AMC 12A (Problems • Answer Key • Resources) | |
Preceded by Problem 23 |
Followed by Problem 25 |
1 • 2 • 3 • 4 • 5 • 6 • 7 • 8 • 9 • 10 • 11 • 12 • 13 • 14 • 15 • 16 • 17 • 18 • 19 • 20 • 21 • 22 • 23 • 24 • 25 | |
All AMC 12 Problems and Solutions |
The problems on this page are copyrighted by the Mathematical Association of America's American Mathematics Competitions.